mirror of
https://github.com/MartinThoma/LaTeX-examples.git
synced 2025-04-26 06:48:04 +02:00
95 lines
4.3 KiB
TeX
95 lines
4.3 KiB
TeX
\chapter*{Lösungen der Übungsaufgaben}
|
|
\addcontentsline{toc}{chapter}{Lösungen der Übungsaufgaben}
|
|
\begin{solution}[\ref{ub1:aufg1}]
|
|
\textbf{Teilaufgabe a)} Es gilt:
|
|
\begin{enumerate}[label=(\roman*)]
|
|
\item $\emptyset, X \in \fT_X$.
|
|
\item $\fT_X$ ist offensichtlich unter Durchschnitten abgeschlossen,
|
|
d.~h. es gilt für alle $U_1, U_2 \in \fT_X: U_1 \cap U_2 \in \fT_X$.
|
|
\item Auch unter beliebigen Vereinigungen ist $\fT_X$ abgeschlossen,
|
|
d.~h. es gilt für eine beliebige Indexmenge $I$ und alle
|
|
$U_i \in \fT_X$ für alle $i \in I: \bigcup_{i \in I} U_i \in \fT_X$
|
|
\end{enumerate}
|
|
|
|
Also ist $(X, \fT_X)$ ein topologischer Raum.
|
|
|
|
\textbf{Teilaufgabe b)} Wähle $x=1, y=0$. Dann gilt $x \neq y$
|
|
und die einzige Umgebung von $x$ ist $X$. Da $y=0 \in X$ können
|
|
also $x$ und $y$ nicht durch offene Mengen getrennt werden.
|
|
$(X, \fT_X)$ ist also nicht hausdorffsch.
|
|
|
|
\textbf{Teilaufgabe c)} Nach Bemerkung \ref{Trennungseigenschaft}
|
|
sind metrische Räume hausdorffsch. Da $(X, \fT_X)$ nach (b) nicht
|
|
hausdorffsch ist, liefert die Kontraposition der Trennungseigenschaft,
|
|
dass $(X, \fT_X)$ kein metrischer Raum sein kann.
|
|
\end{solution}
|
|
|
|
\begin{solution}[\ref{ub1:aufg4}]
|
|
\textbf{Teilaufgabe a)}
|
|
|
|
\textbf{Beh.:} $\forall a \in \mdz: \Set{a}$ ist abgeschlossen.
|
|
|
|
Sei $a \in \mdz$ beliebig. Dann gilt:
|
|
\todo[inline]{Hat jemand diesen Beweis?}
|
|
|
|
\textbf{Teilaufgabe b)}
|
|
|
|
\textbf{Beh.:} $\Set{-1, 1}$ ist nicht offen
|
|
|
|
\textbf{Bew.:} durch Widerspruch
|
|
|
|
Annahme: $\Set{-1, 1}$ ist offen.
|
|
|
|
Dann gibt es $T \subseteq \fB$, sodass $\bigcup_{M \in T} M = \Set{-1, 1}$.
|
|
Aber alle $U \in \fB$ haben unendlich viele Elemente. Auch endlich
|
|
viele Schnitte von Elementen in $\fB$ haben unendlich viele
|
|
Elemente $\Rightarrow$ keine endliche nicht-leere Menge kann
|
|
in dieser Topologie offen sein $\Rightarrow \Set{-1,1}$ ist
|
|
nicht offen. $\qed$
|
|
|
|
\textbf{Beh.:} Es gibt unendlich viele Primzahlen.
|
|
|
|
\textbf{Bew.:} durch Widerspruch
|
|
|
|
Annahme: Es gibt nur endlich viele Primzahlen $p \in \mdp$
|
|
|
|
Dann ist
|
|
\[\mdz \setminus \Set{-1, +1} \overset{\text{FS d. Arithmetik}}= \bigcup_{p \in \mdp} U_{0,p}\]
|
|
endlich. Das ist ein Widerspruch zu $|\mdz|$ ist unendlich und
|
|
$|\Set{-1,1}|$ ist endlich. $\qed$
|
|
\end{solution}
|
|
|
|
\begin{solution}[\ref{ub2:aufg4}]
|
|
\begin{enumerate}[label=(\alph*)]
|
|
\item \underline{Beh.:} Die offenen Mengen von $P$ sind
|
|
Vereinigungen von Mengen der Form
|
|
\[\prod_{j \in J} U_j \times \prod_{i \in \mdn, i \neq j} P_i\]
|
|
wobei $J \subseteq \mdn$ endlich und $U_j \subseteq P_j$
|
|
offen ist.
|
|
\begin{beweis}
|
|
Nach Definition der Produkttopologie bilden Mengen
|
|
der Form
|
|
\[\prod_{i \in J} U_j \times \prod_{\stackrel{i \in \mdn}{i \notin J}} P_i, \text{ wobei } J \subseteq \mdn \text{ endlich und } U_j \subseteq P_j \text{offen } \forall{j \in J}\]
|
|
eine Basis der Topologie. Damit sind die offenen
|
|
Mengen von $P$ Vereinigungen von Mengen der obigen
|
|
Form. $\qed$
|
|
\end{beweis}
|
|
\item \underline{Beh.:} Die Zusammenhangskomponenten von $P$
|
|
sind alle einpunktig.\xindex{Total Unzusammenhängend}
|
|
\begin{beweis}
|
|
Es seinen $x,y \in P$ und $x$ sowie $y$ liegen in der
|
|
gleichen Zusammenhangskomponente $Z \subseteq P$.
|
|
Da $Z$ zusammenhängend ist und $\forall{i \in I}: p_i : P \rightarrow P_i$
|
|
ist stetig, ist $p_i(Z) \subseteq P_i$ zusammenhängend
|
|
für alle $i \in \mdn$. Die zusammenhängenden Mengen
|
|
von $P_i$ sind genau $\Set{0}$ und $\Set{1}$, d.~h.
|
|
für alle $i \in \mdn$ gilt entweder $p_i(Z) \subseteq \Set{0}$
|
|
oder $p_i(Z) \subseteq \Set{1}$. Es sei $z_i \in \Set{0,1}$
|
|
so, dass $p_i(Z) \subseteq \Set{z_i}$ für alle $i \in \mdn$.
|
|
Dann gilt also:
|
|
\[\underbrace{p_i(x)}_{= x_i} = z_i = \underbrace{p_i(y)}_{= y_i} \forall i \in \mdn\]
|
|
Somit folgt: $x = y \qed$
|
|
|
|
\end{beweis}
|
|
\end{enumerate}
|
|
\end{solution}
|